Monday 27 June 2016

homework and exercises - Equivalent resistance in ladder circuit



I have stumbled upon a given question I really have a hard time to solve. Basically I need to find an equivalent resistance in some form of "ladder" configuration. Where the chain is an infinite sequence of resistors.


ladder


I have really no good idea how to find this equivalent resistance. Trying the old fashioned rule of parallel and resistors in series I came to a very messy formula: $$R + \frac{1}{\frac{1}{R} + \frac{1}{R+\frac{1}{\frac{1}{R}+....}}}$$


I know however that the solution should be much more simple.


Now I tried using Kirchhoff's loop rule. Which states that the power difference in a closed loop must be 0. Naming the "potential currents" between AB $I_1$, between BC (through the single resistance) $I_2$ and the current "from B to the right" $I_3$ Considering the loop containing BC & the rest of the structure this rewrites to: $$I_1 = I_2 + I_3$$ $$I_3 \cdot R - I_2 \cdot R_{eq} = 0$$


The problem is, 2 variables, 2 functions doesn't really bring me closer to an answer :(. What am I missing?




Answer



HINT:


Notice that $$R_{eq}=R+\frac1{\frac1R+\frac1{R_{eq}}}$$


No comments:

Post a Comment

Understanding Stagnation point in pitot fluid

What is stagnation point in fluid mechanics. At the open end of the pitot tube the velocity of the fluid becomes zero.But that should result...